wu :: forums (http://www.ocf.berkeley.edu/~wwu/cgi-bin/yabb/YaBB.cgi)
general >> complex analysis >> Re: Maximum on noncompact set
(Message started by: Eigenray on Apr 9th, 2009, 6:27pm)

Title: Re: Maximum on noncompact set
Post by Eigenray on Apr 9th, 2009, 6:27pm
You mean f extends continuously to the boundary?  (This isn't necessarily true.)

Consider g(z) = f(1/z).  This is clearly analytic inside B(0,1/a), except possibly at 0.  But show it's analytic there too.



Powered by YaBB 1 Gold - SP 1.4!
Forum software copyright © 2000-2004 Yet another Bulletin Board